Principle Questions Question 51

In 1992, there were over 250 rescues of mountain climbers, costing the government almost 3 million dollars. More than...

LizD on August 17, 2018

Confused?

I'm a confused why the answer is c? Could someone please explain it to me? Thank you in advance.

Replies
Create a free account to read and take part in forum discussions.

Already have an account? log in

Allie on January 19, 2019

I also thought these answer choices were a bit confusing and would appreciate an explanation

Ravi on January 20, 2019

@LizD and @Allie,

Happy to help.

This question is a strengthen question in which four of the answers
strengthen, but one doesn't.

The stimulus tells us that in 1992, there were over 250 rescues of
mountain climbers, which cost the government almost 3 million dollars.
We're then told that lots of new climbers enter each year and that
over 25 people passed away in climbing accidents in 1992.

We're then given info that members of a task force have proposed a
bonding arrangement that'd force all climbers to post a large sum of
money to be forfeited to the government in the case of calamity.

The question stem asks us to pick the answer that would not help
support the task force members' proposal. The best way to attack this
question is to view four wrong answers as strengtheners and the right
answer as the answer that either weakens or is irrelevant to the
argument.

We know that the gist of the task force members' proposal is that the
people who are responsible for putting themselves at risk should be
the ones bearing the costs of the risk.

(A) is incorrect because it strengthens the argument. If taxpayers
shouldn't subsidize a freely chosen hobby, then the argument is more
credible, so this answer is out.

(B) is incorrect because if the government is obliged to take measures
to deter people from risking their lives, this supports the task
members' proposal of a bonding arrangement that requires all climbers
to post a large sum of money to be forfeited to the government if a
calamity occurs. This answer strengthens the argument, so it's out.

(C) is correct because it does not strengthen the argument. The
stimulus makes no mention of issuing permits to people, so this answer
choice is dealing with a topic that is not discussed in the argument.
Since it's irrelevant—and we're looking for the answer that DOES NOT
strengthen—this is our choice.

(D) is incorrect because if citizens who use publicly subsidized
rescue services should be required to pay more toward the cost of
these services than citizens who do not, then the task force members'
proposal sounds more reasonable and makes sense as a solution. Since
this strengthens, it's out.

(E) is incorrect because if people who engage in physically risky
behavior that's not essential to anyone's welfare should be held
responsible for the cost of treating any resulting injuries, then the
task force members' proposal makes more sense and sounds like a good
solution. This answer strengthens, so it's out as well.

Does this help? Let us know if you have any more questions!